Clarification of the wording of a statistics problem

  • Thread starter Thread starter Biochemgirl2002
  • Start date Start date
  • Tags Tags
    Statistics
Click For Summary
The discussion focuses on the interpretation of a statistics problem regarding the mean weight differing from 20g. It clarifies that the scientists are investigating whether the population's mean weight is specifically 20g, rather than whether it falls within a range of 1.5g to 41.5g. The task involves constructing a confidence interval around the sample mean to determine if it includes the value of 20g. The key point is that the question is about the mean weight being different from 20g, not about the range of weights. Understanding this distinction is crucial for addressing the problem correctly.
Biochemgirl2002
Messages
29
Reaction score
1
Homework Statement
A sample of 100 flounders of a certain species has sample mean weight 21.5 grams and standard
deviation 8.4. Scientists want to determine if the mean weight differs from 20 grams.
a) At α = 0.05, is there enough evidence to conclude that the weight is different from 20? Use the p-value method.
Relevant Equations
i have no clue
i don't know what to do, but i am not asking help for the whole thing i am just wondering what it means by mean weight differs from 20g? from 1.5g-41.5g? or does it mean when the weight is not 20g
 
Physics news on Phys.org
I think they want you to construct a confidence interval about the sample mean and see if it includes the value 20.
 
rhiana said:
i am just wondering what it means by mean weight differs from 20g? from 1.5g-41.5g? or does it mean when the weight is not 20g

The scientists are interested in whether the mean weight of the population is (or is not) 20g. Their question is not whether the mean weight of the population is in the interval [1.5g, 41.5g].
 
Question: A clock's minute hand has length 4 and its hour hand has length 3. What is the distance between the tips at the moment when it is increasing most rapidly?(Putnam Exam Question) Answer: Making assumption that both the hands moves at constant angular velocities, the answer is ## \sqrt{7} .## But don't you think this assumption is somewhat doubtful and wrong?

Similar threads

Replies
4
Views
2K
  • · Replies 7 ·
Replies
7
Views
1K
  • · Replies 1 ·
Replies
1
Views
1K
  • · Replies 1 ·
Replies
1
Views
2K
  • · Replies 24 ·
Replies
24
Views
6K
  • · Replies 5 ·
Replies
5
Views
3K
  • · Replies 2 ·
Replies
2
Views
2K
  • · Replies 19 ·
Replies
19
Views
1K
Replies
11
Views
3K
  • · Replies 6 ·
Replies
6
Views
2K